trace of symmetric matrix problems












3












$begingroup$


I have the two problems below from a practice exam. I can prove them on their own but am not exactly sure if/how to show that they only hold for symmetric matrices and for '3)' showing that it only holds for a matrix with only positive eigenvalues. I know that if the eigenvalues are all positive the determinant will be positive and the trace but cant see how that affects whether '3)' is true or not.




  1. Show that $operatorname{Tr}(A^2) leq operatorname{Tr}(A)^2$ holds for any symmetric matrix $A$ whose eigenvalues are all non-negative.

  2. Show that $operatorname{Tr}(AB)^2 le operatorname{Tr}(A^2)operatorname{Tr}(B^2)$ holds for any symmetric matrices $A$ and $B$.










share|cite|improve this question











$endgroup$








  • 1




    $begingroup$
    If $lambda_1, ldots, lambda_n$ are the eigenvalues of $A$, what are the eigenvalues of $A^2$? And how the trace of a matrix is related to the eigenvalues?
    $endgroup$
    – thanasissdr
    Oct 28 '15 at 9:08










  • $begingroup$
    for both questions i just worked through an example which proved the questions right. I understand the eigenvalues for $A^2$ are the same as for $A$ but squared and when you add the trace you should get the same value as the eigenvalues added. I just wasn't sure in the second question as it says about all non-negative eigenvalues if just working through an example that proves it right really proves it for all cases and therefore had to go about it a different way
    $endgroup$
    – dmnte
    Oct 28 '15 at 12:25










  • $begingroup$
    for the fourth question I cant really prove it as much as I just show that it is true through a worked example, im guessing this is wrong but im not sure how else to do it
    $endgroup$
    – dmnte
    Oct 28 '15 at 14:49
















3












$begingroup$


I have the two problems below from a practice exam. I can prove them on their own but am not exactly sure if/how to show that they only hold for symmetric matrices and for '3)' showing that it only holds for a matrix with only positive eigenvalues. I know that if the eigenvalues are all positive the determinant will be positive and the trace but cant see how that affects whether '3)' is true or not.




  1. Show that $operatorname{Tr}(A^2) leq operatorname{Tr}(A)^2$ holds for any symmetric matrix $A$ whose eigenvalues are all non-negative.

  2. Show that $operatorname{Tr}(AB)^2 le operatorname{Tr}(A^2)operatorname{Tr}(B^2)$ holds for any symmetric matrices $A$ and $B$.










share|cite|improve this question











$endgroup$








  • 1




    $begingroup$
    If $lambda_1, ldots, lambda_n$ are the eigenvalues of $A$, what are the eigenvalues of $A^2$? And how the trace of a matrix is related to the eigenvalues?
    $endgroup$
    – thanasissdr
    Oct 28 '15 at 9:08










  • $begingroup$
    for both questions i just worked through an example which proved the questions right. I understand the eigenvalues for $A^2$ are the same as for $A$ but squared and when you add the trace you should get the same value as the eigenvalues added. I just wasn't sure in the second question as it says about all non-negative eigenvalues if just working through an example that proves it right really proves it for all cases and therefore had to go about it a different way
    $endgroup$
    – dmnte
    Oct 28 '15 at 12:25










  • $begingroup$
    for the fourth question I cant really prove it as much as I just show that it is true through a worked example, im guessing this is wrong but im not sure how else to do it
    $endgroup$
    – dmnte
    Oct 28 '15 at 14:49














3












3








3


1



$begingroup$


I have the two problems below from a practice exam. I can prove them on their own but am not exactly sure if/how to show that they only hold for symmetric matrices and for '3)' showing that it only holds for a matrix with only positive eigenvalues. I know that if the eigenvalues are all positive the determinant will be positive and the trace but cant see how that affects whether '3)' is true or not.




  1. Show that $operatorname{Tr}(A^2) leq operatorname{Tr}(A)^2$ holds for any symmetric matrix $A$ whose eigenvalues are all non-negative.

  2. Show that $operatorname{Tr}(AB)^2 le operatorname{Tr}(A^2)operatorname{Tr}(B^2)$ holds for any symmetric matrices $A$ and $B$.










share|cite|improve this question











$endgroup$




I have the two problems below from a practice exam. I can prove them on their own but am not exactly sure if/how to show that they only hold for symmetric matrices and for '3)' showing that it only holds for a matrix with only positive eigenvalues. I know that if the eigenvalues are all positive the determinant will be positive and the trace but cant see how that affects whether '3)' is true or not.




  1. Show that $operatorname{Tr}(A^2) leq operatorname{Tr}(A)^2$ holds for any symmetric matrix $A$ whose eigenvalues are all non-negative.

  2. Show that $operatorname{Tr}(AB)^2 le operatorname{Tr}(A^2)operatorname{Tr}(B^2)$ holds for any symmetric matrices $A$ and $B$.







linear-algebra matrices trace






share|cite|improve this question















share|cite|improve this question













share|cite|improve this question




share|cite|improve this question








edited Oct 29 '15 at 7:14









thanasissdr

5,55111325




5,55111325










asked Oct 28 '15 at 9:03









dmntedmnte

1129




1129








  • 1




    $begingroup$
    If $lambda_1, ldots, lambda_n$ are the eigenvalues of $A$, what are the eigenvalues of $A^2$? And how the trace of a matrix is related to the eigenvalues?
    $endgroup$
    – thanasissdr
    Oct 28 '15 at 9:08










  • $begingroup$
    for both questions i just worked through an example which proved the questions right. I understand the eigenvalues for $A^2$ are the same as for $A$ but squared and when you add the trace you should get the same value as the eigenvalues added. I just wasn't sure in the second question as it says about all non-negative eigenvalues if just working through an example that proves it right really proves it for all cases and therefore had to go about it a different way
    $endgroup$
    – dmnte
    Oct 28 '15 at 12:25










  • $begingroup$
    for the fourth question I cant really prove it as much as I just show that it is true through a worked example, im guessing this is wrong but im not sure how else to do it
    $endgroup$
    – dmnte
    Oct 28 '15 at 14:49














  • 1




    $begingroup$
    If $lambda_1, ldots, lambda_n$ are the eigenvalues of $A$, what are the eigenvalues of $A^2$? And how the trace of a matrix is related to the eigenvalues?
    $endgroup$
    – thanasissdr
    Oct 28 '15 at 9:08










  • $begingroup$
    for both questions i just worked through an example which proved the questions right. I understand the eigenvalues for $A^2$ are the same as for $A$ but squared and when you add the trace you should get the same value as the eigenvalues added. I just wasn't sure in the second question as it says about all non-negative eigenvalues if just working through an example that proves it right really proves it for all cases and therefore had to go about it a different way
    $endgroup$
    – dmnte
    Oct 28 '15 at 12:25










  • $begingroup$
    for the fourth question I cant really prove it as much as I just show that it is true through a worked example, im guessing this is wrong but im not sure how else to do it
    $endgroup$
    – dmnte
    Oct 28 '15 at 14:49








1




1




$begingroup$
If $lambda_1, ldots, lambda_n$ are the eigenvalues of $A$, what are the eigenvalues of $A^2$? And how the trace of a matrix is related to the eigenvalues?
$endgroup$
– thanasissdr
Oct 28 '15 at 9:08




$begingroup$
If $lambda_1, ldots, lambda_n$ are the eigenvalues of $A$, what are the eigenvalues of $A^2$? And how the trace of a matrix is related to the eigenvalues?
$endgroup$
– thanasissdr
Oct 28 '15 at 9:08












$begingroup$
for both questions i just worked through an example which proved the questions right. I understand the eigenvalues for $A^2$ are the same as for $A$ but squared and when you add the trace you should get the same value as the eigenvalues added. I just wasn't sure in the second question as it says about all non-negative eigenvalues if just working through an example that proves it right really proves it for all cases and therefore had to go about it a different way
$endgroup$
– dmnte
Oct 28 '15 at 12:25




$begingroup$
for both questions i just worked through an example which proved the questions right. I understand the eigenvalues for $A^2$ are the same as for $A$ but squared and when you add the trace you should get the same value as the eigenvalues added. I just wasn't sure in the second question as it says about all non-negative eigenvalues if just working through an example that proves it right really proves it for all cases and therefore had to go about it a different way
$endgroup$
– dmnte
Oct 28 '15 at 12:25












$begingroup$
for the fourth question I cant really prove it as much as I just show that it is true through a worked example, im guessing this is wrong but im not sure how else to do it
$endgroup$
– dmnte
Oct 28 '15 at 14:49




$begingroup$
for the fourth question I cant really prove it as much as I just show that it is true through a worked example, im guessing this is wrong but im not sure how else to do it
$endgroup$
– dmnte
Oct 28 '15 at 14:49










2 Answers
2






active

oldest

votes


















1












$begingroup$

About question $#4$:



Notation: Let $C= AB$ and $K^2_{ii}$ denote the element at the $(i,i)$ position of the matrix $K^2$.



Firstly, due to symmetry of the $ntimes n$ matrices $A,B$, it is easy to prove that:
$$c_{ii} le left(A^2_{ii}right)^{1/2} cdot left(B^2_{ii}right)^{1/2},quad i = 1,ldots, n.tag 1$$





Proof of $(1)$



$c_{ii} =^color{red}{starstar} a_{i1} cdot b_{i1} + cdots +a_{in}cdot b_{in}= sumlimits_{j=1}^{n}a_{ij}cdot b_{ij}color{blue}{le^star} left(sumlimits_{j=1}^{n}a^2_{ij}right)^{1/2}cdot left(sumlimits_{j=1}^nb_{ij}^2right)^{1/2}=left(A^2_{ii}right)^{1/2}cdot left(B^2_{ii}right)^{1/2}quad text{QED}$



Thus, we have:
$$big[operatorname{trace} (AB)big]^2=left(sum_{i=1}^n c_{ii}right)^2leleft[ sum_{i=1}^n left(A^2_{ii}right)^{1/2}cdot left(B^2_{ii}right)^{1/2}right]^2color{blue}{le^star}sum_{i=1}^n A^2_{ii} cdot sum_{i=1}^n B^2_{ii}=operatorname{trace} (A^2) cdot operatorname{trace} (B^2)$$





$^color{blue}{star}$ We have applied the Cauchy-Schwarz inequality.



$^color{red}{starstar}$ Normally, it is:
$c_{ii} = a_{i1}b_{1i} + a_{i2}b_{2i} +cdots + a_{in}b_{ni},$
but notice that $b_{kell} = b_{ell k}$.






share|cite|improve this answer











$endgroup$





















    1












    $begingroup$

    Question No.3 is more related to the fact that for given numbers $x_1,dots,x_n$, the following inequality $$x_1^2+dots+x_n^2leq ,(x_1+dots+x_n)^2$$ holds only if $x_i$ are non-negative. Let $A$ be any diagonalizable matrix so that $A=TLambda T^{-1}$ and $A^2=TLambda^2 T^{-1}$. Thus, if $x_1,dots,x_n$ are the eigenvalues, then $mathrm{trace}(A^2)=x_1^2+dots+x_n^2 $ and $mathrm{trace}(A)^2=(x_1+dots+x_n)^2 $. Note that symmetric matrices are readily diagonalizable since they are normal.



    Question No.4 is more related to the fact that trace is an inner product in the space of symmetric matrices. In fact, that inequality you have given is Cauchy-Schwartz indeed.






    share|cite|improve this answer











    $endgroup$













      Your Answer





      StackExchange.ifUsing("editor", function () {
      return StackExchange.using("mathjaxEditing", function () {
      StackExchange.MarkdownEditor.creationCallbacks.add(function (editor, postfix) {
      StackExchange.mathjaxEditing.prepareWmdForMathJax(editor, postfix, [["$", "$"], ["\\(","\\)"]]);
      });
      });
      }, "mathjax-editing");

      StackExchange.ready(function() {
      var channelOptions = {
      tags: "".split(" "),
      id: "69"
      };
      initTagRenderer("".split(" "), "".split(" "), channelOptions);

      StackExchange.using("externalEditor", function() {
      // Have to fire editor after snippets, if snippets enabled
      if (StackExchange.settings.snippets.snippetsEnabled) {
      StackExchange.using("snippets", function() {
      createEditor();
      });
      }
      else {
      createEditor();
      }
      });

      function createEditor() {
      StackExchange.prepareEditor({
      heartbeatType: 'answer',
      autoActivateHeartbeat: false,
      convertImagesToLinks: true,
      noModals: true,
      showLowRepImageUploadWarning: true,
      reputationToPostImages: 10,
      bindNavPrevention: true,
      postfix: "",
      imageUploader: {
      brandingHtml: "Powered by u003ca class="icon-imgur-white" href="https://imgur.com/"u003eu003c/au003e",
      contentPolicyHtml: "User contributions licensed under u003ca href="https://creativecommons.org/licenses/by-sa/3.0/"u003ecc by-sa 3.0 with attribution requiredu003c/au003e u003ca href="https://stackoverflow.com/legal/content-policy"u003e(content policy)u003c/au003e",
      allowUrls: true
      },
      noCode: true, onDemand: true,
      discardSelector: ".discard-answer"
      ,immediatelyShowMarkdownHelp:true
      });


      }
      });














      draft saved

      draft discarded


















      StackExchange.ready(
      function () {
      StackExchange.openid.initPostLogin('.new-post-login', 'https%3a%2f%2fmath.stackexchange.com%2fquestions%2f1501499%2ftrace-of-symmetric-matrix-problems%23new-answer', 'question_page');
      }
      );

      Post as a guest















      Required, but never shown

























      2 Answers
      2






      active

      oldest

      votes








      2 Answers
      2






      active

      oldest

      votes









      active

      oldest

      votes






      active

      oldest

      votes









      1












      $begingroup$

      About question $#4$:



      Notation: Let $C= AB$ and $K^2_{ii}$ denote the element at the $(i,i)$ position of the matrix $K^2$.



      Firstly, due to symmetry of the $ntimes n$ matrices $A,B$, it is easy to prove that:
      $$c_{ii} le left(A^2_{ii}right)^{1/2} cdot left(B^2_{ii}right)^{1/2},quad i = 1,ldots, n.tag 1$$





      Proof of $(1)$



      $c_{ii} =^color{red}{starstar} a_{i1} cdot b_{i1} + cdots +a_{in}cdot b_{in}= sumlimits_{j=1}^{n}a_{ij}cdot b_{ij}color{blue}{le^star} left(sumlimits_{j=1}^{n}a^2_{ij}right)^{1/2}cdot left(sumlimits_{j=1}^nb_{ij}^2right)^{1/2}=left(A^2_{ii}right)^{1/2}cdot left(B^2_{ii}right)^{1/2}quad text{QED}$



      Thus, we have:
      $$big[operatorname{trace} (AB)big]^2=left(sum_{i=1}^n c_{ii}right)^2leleft[ sum_{i=1}^n left(A^2_{ii}right)^{1/2}cdot left(B^2_{ii}right)^{1/2}right]^2color{blue}{le^star}sum_{i=1}^n A^2_{ii} cdot sum_{i=1}^n B^2_{ii}=operatorname{trace} (A^2) cdot operatorname{trace} (B^2)$$





      $^color{blue}{star}$ We have applied the Cauchy-Schwarz inequality.



      $^color{red}{starstar}$ Normally, it is:
      $c_{ii} = a_{i1}b_{1i} + a_{i2}b_{2i} +cdots + a_{in}b_{ni},$
      but notice that $b_{kell} = b_{ell k}$.






      share|cite|improve this answer











      $endgroup$


















        1












        $begingroup$

        About question $#4$:



        Notation: Let $C= AB$ and $K^2_{ii}$ denote the element at the $(i,i)$ position of the matrix $K^2$.



        Firstly, due to symmetry of the $ntimes n$ matrices $A,B$, it is easy to prove that:
        $$c_{ii} le left(A^2_{ii}right)^{1/2} cdot left(B^2_{ii}right)^{1/2},quad i = 1,ldots, n.tag 1$$





        Proof of $(1)$



        $c_{ii} =^color{red}{starstar} a_{i1} cdot b_{i1} + cdots +a_{in}cdot b_{in}= sumlimits_{j=1}^{n}a_{ij}cdot b_{ij}color{blue}{le^star} left(sumlimits_{j=1}^{n}a^2_{ij}right)^{1/2}cdot left(sumlimits_{j=1}^nb_{ij}^2right)^{1/2}=left(A^2_{ii}right)^{1/2}cdot left(B^2_{ii}right)^{1/2}quad text{QED}$



        Thus, we have:
        $$big[operatorname{trace} (AB)big]^2=left(sum_{i=1}^n c_{ii}right)^2leleft[ sum_{i=1}^n left(A^2_{ii}right)^{1/2}cdot left(B^2_{ii}right)^{1/2}right]^2color{blue}{le^star}sum_{i=1}^n A^2_{ii} cdot sum_{i=1}^n B^2_{ii}=operatorname{trace} (A^2) cdot operatorname{trace} (B^2)$$





        $^color{blue}{star}$ We have applied the Cauchy-Schwarz inequality.



        $^color{red}{starstar}$ Normally, it is:
        $c_{ii} = a_{i1}b_{1i} + a_{i2}b_{2i} +cdots + a_{in}b_{ni},$
        but notice that $b_{kell} = b_{ell k}$.






        share|cite|improve this answer











        $endgroup$
















          1












          1








          1





          $begingroup$

          About question $#4$:



          Notation: Let $C= AB$ and $K^2_{ii}$ denote the element at the $(i,i)$ position of the matrix $K^2$.



          Firstly, due to symmetry of the $ntimes n$ matrices $A,B$, it is easy to prove that:
          $$c_{ii} le left(A^2_{ii}right)^{1/2} cdot left(B^2_{ii}right)^{1/2},quad i = 1,ldots, n.tag 1$$





          Proof of $(1)$



          $c_{ii} =^color{red}{starstar} a_{i1} cdot b_{i1} + cdots +a_{in}cdot b_{in}= sumlimits_{j=1}^{n}a_{ij}cdot b_{ij}color{blue}{le^star} left(sumlimits_{j=1}^{n}a^2_{ij}right)^{1/2}cdot left(sumlimits_{j=1}^nb_{ij}^2right)^{1/2}=left(A^2_{ii}right)^{1/2}cdot left(B^2_{ii}right)^{1/2}quad text{QED}$



          Thus, we have:
          $$big[operatorname{trace} (AB)big]^2=left(sum_{i=1}^n c_{ii}right)^2leleft[ sum_{i=1}^n left(A^2_{ii}right)^{1/2}cdot left(B^2_{ii}right)^{1/2}right]^2color{blue}{le^star}sum_{i=1}^n A^2_{ii} cdot sum_{i=1}^n B^2_{ii}=operatorname{trace} (A^2) cdot operatorname{trace} (B^2)$$





          $^color{blue}{star}$ We have applied the Cauchy-Schwarz inequality.



          $^color{red}{starstar}$ Normally, it is:
          $c_{ii} = a_{i1}b_{1i} + a_{i2}b_{2i} +cdots + a_{in}b_{ni},$
          but notice that $b_{kell} = b_{ell k}$.






          share|cite|improve this answer











          $endgroup$



          About question $#4$:



          Notation: Let $C= AB$ and $K^2_{ii}$ denote the element at the $(i,i)$ position of the matrix $K^2$.



          Firstly, due to symmetry of the $ntimes n$ matrices $A,B$, it is easy to prove that:
          $$c_{ii} le left(A^2_{ii}right)^{1/2} cdot left(B^2_{ii}right)^{1/2},quad i = 1,ldots, n.tag 1$$





          Proof of $(1)$



          $c_{ii} =^color{red}{starstar} a_{i1} cdot b_{i1} + cdots +a_{in}cdot b_{in}= sumlimits_{j=1}^{n}a_{ij}cdot b_{ij}color{blue}{le^star} left(sumlimits_{j=1}^{n}a^2_{ij}right)^{1/2}cdot left(sumlimits_{j=1}^nb_{ij}^2right)^{1/2}=left(A^2_{ii}right)^{1/2}cdot left(B^2_{ii}right)^{1/2}quad text{QED}$



          Thus, we have:
          $$big[operatorname{trace} (AB)big]^2=left(sum_{i=1}^n c_{ii}right)^2leleft[ sum_{i=1}^n left(A^2_{ii}right)^{1/2}cdot left(B^2_{ii}right)^{1/2}right]^2color{blue}{le^star}sum_{i=1}^n A^2_{ii} cdot sum_{i=1}^n B^2_{ii}=operatorname{trace} (A^2) cdot operatorname{trace} (B^2)$$





          $^color{blue}{star}$ We have applied the Cauchy-Schwarz inequality.



          $^color{red}{starstar}$ Normally, it is:
          $c_{ii} = a_{i1}b_{1i} + a_{i2}b_{2i} +cdots + a_{in}b_{ni},$
          but notice that $b_{kell} = b_{ell k}$.







          share|cite|improve this answer














          share|cite|improve this answer



          share|cite|improve this answer








          edited Oct 28 '15 at 21:59

























          answered Oct 28 '15 at 19:16









          thanasissdrthanasissdr

          5,55111325




          5,55111325























              1












              $begingroup$

              Question No.3 is more related to the fact that for given numbers $x_1,dots,x_n$, the following inequality $$x_1^2+dots+x_n^2leq ,(x_1+dots+x_n)^2$$ holds only if $x_i$ are non-negative. Let $A$ be any diagonalizable matrix so that $A=TLambda T^{-1}$ and $A^2=TLambda^2 T^{-1}$. Thus, if $x_1,dots,x_n$ are the eigenvalues, then $mathrm{trace}(A^2)=x_1^2+dots+x_n^2 $ and $mathrm{trace}(A)^2=(x_1+dots+x_n)^2 $. Note that symmetric matrices are readily diagonalizable since they are normal.



              Question No.4 is more related to the fact that trace is an inner product in the space of symmetric matrices. In fact, that inequality you have given is Cauchy-Schwartz indeed.






              share|cite|improve this answer











              $endgroup$


















                1












                $begingroup$

                Question No.3 is more related to the fact that for given numbers $x_1,dots,x_n$, the following inequality $$x_1^2+dots+x_n^2leq ,(x_1+dots+x_n)^2$$ holds only if $x_i$ are non-negative. Let $A$ be any diagonalizable matrix so that $A=TLambda T^{-1}$ and $A^2=TLambda^2 T^{-1}$. Thus, if $x_1,dots,x_n$ are the eigenvalues, then $mathrm{trace}(A^2)=x_1^2+dots+x_n^2 $ and $mathrm{trace}(A)^2=(x_1+dots+x_n)^2 $. Note that symmetric matrices are readily diagonalizable since they are normal.



                Question No.4 is more related to the fact that trace is an inner product in the space of symmetric matrices. In fact, that inequality you have given is Cauchy-Schwartz indeed.






                share|cite|improve this answer











                $endgroup$
















                  1












                  1








                  1





                  $begingroup$

                  Question No.3 is more related to the fact that for given numbers $x_1,dots,x_n$, the following inequality $$x_1^2+dots+x_n^2leq ,(x_1+dots+x_n)^2$$ holds only if $x_i$ are non-negative. Let $A$ be any diagonalizable matrix so that $A=TLambda T^{-1}$ and $A^2=TLambda^2 T^{-1}$. Thus, if $x_1,dots,x_n$ are the eigenvalues, then $mathrm{trace}(A^2)=x_1^2+dots+x_n^2 $ and $mathrm{trace}(A)^2=(x_1+dots+x_n)^2 $. Note that symmetric matrices are readily diagonalizable since they are normal.



                  Question No.4 is more related to the fact that trace is an inner product in the space of symmetric matrices. In fact, that inequality you have given is Cauchy-Schwartz indeed.






                  share|cite|improve this answer











                  $endgroup$



                  Question No.3 is more related to the fact that for given numbers $x_1,dots,x_n$, the following inequality $$x_1^2+dots+x_n^2leq ,(x_1+dots+x_n)^2$$ holds only if $x_i$ are non-negative. Let $A$ be any diagonalizable matrix so that $A=TLambda T^{-1}$ and $A^2=TLambda^2 T^{-1}$. Thus, if $x_1,dots,x_n$ are the eigenvalues, then $mathrm{trace}(A^2)=x_1^2+dots+x_n^2 $ and $mathrm{trace}(A)^2=(x_1+dots+x_n)^2 $. Note that symmetric matrices are readily diagonalizable since they are normal.



                  Question No.4 is more related to the fact that trace is an inner product in the space of symmetric matrices. In fact, that inequality you have given is Cauchy-Schwartz indeed.







                  share|cite|improve this answer














                  share|cite|improve this answer



                  share|cite|improve this answer








                  edited Oct 29 '15 at 9:18

























                  answered Oct 29 '15 at 7:33









                  dineshdileepdineshdileep

                  5,96611735




                  5,96611735






























                      draft saved

                      draft discarded




















































                      Thanks for contributing an answer to Mathematics Stack Exchange!


                      • Please be sure to answer the question. Provide details and share your research!

                      But avoid



                      • Asking for help, clarification, or responding to other answers.

                      • Making statements based on opinion; back them up with references or personal experience.


                      Use MathJax to format equations. MathJax reference.


                      To learn more, see our tips on writing great answers.




                      draft saved


                      draft discarded














                      StackExchange.ready(
                      function () {
                      StackExchange.openid.initPostLogin('.new-post-login', 'https%3a%2f%2fmath.stackexchange.com%2fquestions%2f1501499%2ftrace-of-symmetric-matrix-problems%23new-answer', 'question_page');
                      }
                      );

                      Post as a guest















                      Required, but never shown





















































                      Required, but never shown














                      Required, but never shown












                      Required, but never shown







                      Required, but never shown

































                      Required, but never shown














                      Required, but never shown












                      Required, but never shown







                      Required, but never shown







                      Popular posts from this blog

                      Mario Kart Wii

                      What does “Dominus providebit” mean?

                      Antonio Litta Visconti Arese